You are on page 1of 9

ACTS 4302

Instructor: Natalia A. Humphreys


SOLUTION TO HOMEWORK 6
Lesson 11: The Black-Scholes formula: applications and volatility.
Lesson 12: Delta hedging.
Problem 1
For a 1-year European call option on a stock, you are given:
(i) The stocks price is 55.
(ii) The strike price is 65.
(iii) The stocks volatility is 0.25.
(iv) The continuous annual dividend rate of the stock is 3%
(v) The continuously compounded risk-free rate is 5%.
(vi) The option is priced with the Black-Scholes formula.
You purchase this option. Calculate your profit after 3 months if the stock price is 60 then.
Solution. At time T=0 we are given: Eur. call, t = 1, S = 55, K = 65, = 0.25, r = 0.05, = 0.03.
C = Set N (d1 ) Kert N (d2 ), where

ln 55
+ 0.05 0.03 + 0.5 0.252 1
ln (S/K) + (r + 12 2 )t
65

d1 =
=
= 0.4632
t
0.25 1
N (d1 ) = N (0.46) = 1 0.6772 = 0.3228

d2 = d1 t = 0.4632 0.25 = 0.7132


N (d2 ) = N (0.71) = 1 0.7611 = 0.2389
Kert = 65e0.051 = 61.8299
Set = 55e0.03 = 53.3745
C = 53.3745 0.3228 61.8299 0.2389 = 17.2293 14.7712 = 2.4581
At time T=3 months we are given: Eur. call, t = 0.75, S = 60, K = 65, = 0.25, r = 0.05, = 0.03.

+ 0.5 0.252 0.75
ln 60
65 + 0.05 0.03

= 0.1922
d1 =
0.25 0.75
N (d1 ) = N (0.19) = 1 0.5753 = 0.4247

d2 = d1 t = 0.1922 0.2165 = 0.4087


N (d2 ) = N (0.41) = 1 0.6591 = 0.3409
Kert = 65e0.050.75 = 62.6076
Set = 60e0.030.75 = 58.6651
C = 58.6651 0.4247 62.6076 0.3409 = 24.9151 21.3429 = 3.5721
The profit is:
3.5721 2.4581 e0.050.25 = 1.083

Problem 2
For a bear spread, you are given:
(i) The stocks price is 75.
(ii) The stock pays no dividends.
(iii) The stocks volatility is 0.3.
(iv) The bear spread consists of buying a 70-strike European put and selling a 60-strike European put.
(v) Time to expiry of the options is 6 months.
(vi) The continuously compounded risk-free rate is 4%.

ACTS 4302. AU 2014. SOLUTION TO HOMEWORK 6.

Copyright Natalia A. Humphreys, 2014

Calculate your profit on the bear spread after 3 months if the stock price is 73 then.
Solution. At time T=0 we are given: Eur. puts, t = 0.5, S = 75, = 0, = 0.3, r = 0.04.
For a 70-strike put, the value is:
P = Kert N (d2 ) Set N (d1 ), where

2 0.5
ln 75
+
0.04
+
0.5

0.3
ln (S/K) + (r + 12 2 )t
0.1115

d1 =
= 70
=
= 0.5256
0.2121
t
0.3 0.5
N (d1 ) = N (0.53) = 1 0.7019 = 0.2981

d2 = d1 t = 0.5256 0.2121 = 0.3135


N (d2 ) = N (0.31) = 1 0.6217 = 0.3783
Kert = 70e0.040.5 = 68.6139
Set = 75
P = 68.6139 0.3783 75 0.2981 = 25.9566 22.3575 = 3.5991
For a 60-strike put, the value is:

2 0.5
ln 75
+
0.04
+
0.5

0.3
0.2656

d1 = 60
=
= 1.2523
0.2121
0.3 0.5
N (d1 ) = N (1.25) = 1 0.8944 = 0.1056

d2 = d1 t = 1.2523 0.2121 = 1.0401


N (d2 ) = N (1.04) = 1 0.8508 = 0.1492
Kert = 60e0.040.5 = 58.8119
Set = 75
P = 58.8119 0.1492 75 0.1056 = 8.7747 7.92 = 0.8547
Cost of the bear spread:
3.5991 0.8547 = 2.7444
At time T=0.25 we are given: Eur. puts, t = 0.25, S = 73, = 0, = 0.3, r = 0.04.
For a 70-strike put, the value is:

0.5 0.32 0.25
ln 73
70 + 0.04 +

d1 =
= 0.4214
0.3 0.25
N (d1 ) = N (0.42) = 1 0.6628 = 0.3372

d2 = d1 t = 0.4214 0.15 = 0.2714


N (d2 ) = N (0.27) = 1 0.6064 = 0.3936
Kert = 70e0.040.25 = 69.3035
Set = 73
P = 69.3035 0.3936 73 0.3372 = 27.2779 24.6156 = 2.6623
For a 60-strike put, the value is:

ln 73
0.5 0.32 0.25
60 + 0.04 +

d1 =
= 1.4491
0.3 0.25
N (d1 ) = N (1.45) = 1 0.9265 = 0.0735

d2 = d1 t = 1.4491 0.15 = 1.2991


N (d2 ) = N (1.3) = 1 0.9032 = 0.0968
Kert = 60e0.040.25 = 59.403
Set = 73
P = 59.403 0.0968 73 0.0735 = 5.7502 5.3655 = 0.3847
Page 2 of 9

ACTS 4302. AU 2014. SOLUTION TO HOMEWORK 6.

Copyright Natalia A. Humphreys, 2014

Cost of the bear spread:


2.6623 0.3847 = 2.2775
The profit is:
2.2775 2.7444 e0.040.25 = 0.4944

Problem 3
For a European put option on a stock with one year to expiry, you are given:
(i)
(ii)
(iii)
(iv)
(v)

The stocks price is 50.


The strike price is 50.
r=0.04.
= 0.04
The price of the option is 6.22.

Determine the implied volatility of the stock using the Black-Scholes formula.
Solution. We are given: Eur. put, t = 1, S = 50, K = 50, = 0.04, r = 0.04, P = 6.22.
P = Kert N (d2 ) Set N (d1 ), where

ln (S/K) + (r + 12 2 )t

d1 =
= 0.5 t = 0.5
t
N (d1 ) = N (0.5) = 1 N (0.5)

d2 = d1 t = 0.5 t t = 0.5
N (d2 ) = N (0.5)
P = Kert N (d2 ) Set (1 N (d2 )) = N (d2 )[Kert + Set ] Set
P + Set
6.22 + 50e0.04
54.2595
=
=
= 0.5647
0.04
rt
t
2 50e
96.0789
Ke
+ Se
0.16
= 0.32 
d2 0.16 d2 = 0.16 = 0.5 =
0.5
N (d2 ) =

Problem 4
For a 91-day European call option on a stock, you are given:
(i)
(ii)
(iii)
(iv)
(v)

The
The
The
The
The

stocks price is 40.


strike price is 45.
continuously compounded risk-free interest rate is 5%.
stock pays no dividends.
stocks annual volatility is 25%

You write the option and delta-hedge it.


Calculate the 1-day mark-to-market profit on a delta-hedged portfolio for this option if the stock price
increases to 45.
Note: Work this problem out using interpolation (rather than rounding) to evaluate cumulative normal
distribution values.
Solution. Recall that the overnight profit on a delta-hedged portfolio has three components:
1. The change in the value of the option.
2. times the change in the price of the stock.
3. Interest on the borrowed money.


Profit = (C(S1 ) C(S0 )) + (S1 S0 ) er/365 1 (S0 C(S0 ))
Page 3 of 9

ACTS 4302. AU 2014. SOLUTION TO HOMEWORK 6.

Copyright Natalia A. Humphreys, 2014

Calculate the original price of a call using t = 91/365.


C = Set N (d1 ) Kert N (d2 ), where
ln 40 + 0.05 0 + 0.5 0.252
ln (S/K) + (r + 12 2 )t

q
d1 =
= 45
91
t
0.25 365

91
365

0.0975
=
0.1248

= 0.7813 = 0.78 0.87 0.79 0.13


N (d1 ) = N (0.7813) = 1 (N (0.78) 0.87 + N (0.79) 0.13) = 1 (0.7823 0.87 + 0.7852 0.13) =
= 1 0.7827 = 0.2173

d2 = d1 t = 0.7813 0.1248 = 0.9061 = 0.90 0.39 0.91 0.61


N (d2 ) = N (0.9061) = 1 (N (0.9) 0.39 + N (0.91) 0.61) = 1 (0.819 0.39 + 0.8186 0.61) =
= 1 0.8175 = 0.1825
91

Kert = 45e0.05 365 = 44.4425


Set = 40
C = 40 0.2173 44.4425 0.1825 = 8.6929 8.1087 = 0.5842
Also,
= et N (d1 ) = N (d1 ) = 0.2173
Calculate the price of a call one day later using t = 90/365.
 90
ln 45
+ 0.5 0.252 365
0.02
45 + 0.05 0q
=
= 0.1614 = 0.16 0.86 + 0.17 0.14
d1 =
0.1241
90
0.25 365
N (d1 ) = N (0.1614) = N (0.16) 0.86 + N (0.17) 0.14 = 0.5636 0.86 + 0.5675 0.14 = 0.5642

d2 = d1 t = 0.1614 0.1241 = 0.0373 = 0.03 0.27 + 0.04 0.73


N (d2 ) = N (0.0373) = N (0.03) 0.27 + N (0.04) 0.73 = 0.512 0.27 + 0.516 0.73 = 0.51492
90

Kert = 45e0.05 365 = 44.4486


Set = 45
C = 45 0.5642 44.4486 0.51492 = 25.389 22.8875 = 2.5015
Thus, the change in the value of the option:
C(S1 ) C(S0 ) = 2.5015 0.5842 = 1.9173
times the change in the price of the stock:
(S1 S0 ) = 0.2173(45 40) = 1.0865
Interest on the borrowed money:




er/365 1 (S0 C(S0 )) = e0.05/365 1 (0.2173 40 0.5842) = 0.0011
Therefore, the total profit is:
Profit = 1.9173 + 1.0865 0.0011 = 0.8319

Problem 5
For 3-month European call options on a stock, you are given:
(i) The stock price is 60.
(ii) The stock pays no dividends.
(iii) The annual volatility is 20%.
(iv) The continuously compounded risk-free interest rate is 8%.
Page 4 of 9

ACTS 4302. AU 2014. SOLUTION TO HOMEWORK 6.

Copyright Natalia A. Humphreys, 2014

A market-maker writes a 55-65 bull spread with calls.


Determine the investment required by the market-maker to write this bull spread and delta-hedge it.
Solution. In a bull spread, the purchaser buys a lower-strike option and sells a higher-strike option.
Therefore, the counterparty, the marker-maker, sells a lower-strike option and purchases a higher-strike
option. Here, he purchases a 65-strike call and sells a 55-strike call. To hedge, he must buy 55 shares
of stock and sell 65 shares of stock.
The premium and delta for the 55-strike call is
C = Set N (d1 ) Kert N (d2 ), where

60
ln 55
+ 0.08 + 0.5 0.22 0.25
ln (S/K) + (r + 21 2 )t
0.1120

d1 =
=
=
= 1.12
0.1
t
0.2 0.25
N (d1 ) = N (1.12) = 0.8686
55 = et N (d1 ) = N (d1 ) = 0.8686

d2 = d1 t = 1.12 0.1 = 1.02


N (d2 ) = N (1.02) = 0.8461
Kert = 55e0.080.25 = 53.9109
Set = 60
C = 60 0.8686 53.9109 0.8461 = 52.116 45.614 = 6.502
The premium and delta for the 65-strike call is

60
+ 0.08 + 0.5 0.22 0.25
ln 65

= 0.5504
d1 =
0.2 0.25
N (d1 ) = N (0.55) = 1 0.7088 = 0.2912
65 = et N (d1 ) = N (d1 ) = 0.2912

d2 = d1 t = 0.5504 0.1 = 0.6504


N (d2 ) = N (0.65) = 1 0.7422 = 0.2578
Kert = 65e0.080.25 = 63.7129
Set = 60
C = 60 0.2912 63.7129 0.2578 = 17.472 16.4252 = 1.0468
To hedge the purchase of a 65-strike call and sale of a 55-strike call, the market-maker must buy 55
shares of stock and sell 65 shares of stock. The total investment is:
S0 (55 65 ) = 60(0.8686 0.2912) = 60 0.5774 = 34.644
Sale of a bull spread will land the market-maker
C55 C65 = 6.502 1.0468 = 5.4552
Hence, the total investment is:
34.644 5.4552 = 29.1888

Problem 6
You are given the following information for a delta-hedged portfolio for a call option on a stock:
(i) The underlying stocks price is 45.
(ii) The continuous annual dividend rate of the stock is 0.025.
(iii) = 0.35
(iv) The options expiry is 1 year.
(v) The continuously compounded risk-free interest rate is 5%.
Page 5 of 9

ACTS 4302. AU 2014. SOLUTION TO HOMEWORK 6.

Copyright Natalia A. Humphreys, 2014

Determine the two stock prices at the end of one week for which there would be approximately no gain
or loss for the delta-hedged portfolio.
Solution. Price movement with no gain or loss to delta-hedger: the money-maker would break even if
the stock moves by about one standard deviation around the mean to either

S + S h or S S h
r

7
S + S h = 45 + 45 0.35
= 47.18
365
r

7
S S h = 45 45 0.35
= 42.82
365

Problem 7
For a 1-year European call option on a stock, you are given:
(i)
(ii)
(iii)
(iv)
(v)

The underlying stocks price is 50.


The strike price is 55.
The continuously compounded risk-free interest rate is 5%.
The continuously compounded dividend yield on the stock is 3%.
For a 1-year European put option on the same stock with a strike price of 55, = .50 and
= 0.03.

Determine the approximate change in the value of the call if the stocks price decreases to 49 instantaneously.
Solution. Delta-gamma-theta approximation
1
C(St+h ) = C(St ) +  + 2 + h
2
Since the stocks price decreases to 49 instantaneously, = 0.
call = put + et = 0.5 + e0.031 = 0.4704
call = put = 0.03

Hence,
C(S1 ) C(S0 ) = 0.4704(1) + 0.5 0.03 1 = 0.4554

Problem 8
For a European call option on a nondividend paying stock with one year to expiry, you are given:
(i)
(ii)
(iii)
(iv)
(v)

The stock follows the Black-Scholes framework.


The underlying stocks price is 60.
The strike price is 70.
The continuously compounded risk-free interest rate is 6%.
Gamma for the option is 0.0524.

The price of the stock jumps to 60.50 and the price of the option increases by 0.0985.
Determine the implied volatility of the stock based on the delta-gamma approximation.
Page 6 of 9

Copyright Natalia A. Humphreys, 2014

ACTS 4302. AU 2014. SOLUTION TO HOMEWORK 6.

Solution.
C(S1 ) C(S0 ) =  + 0.52
0.0985 = call 0.5 + 0.5 0.0524 0.52
call = 0.1839 = et N (d1 ) = N (d1 )


2
ln 60
0.09415 + 0.5 2
70 + 0.06 + 0.5
d1 0.9 =
=

0.5 2 + 0.9 0.09415 = 0


D = 0.92 + 4 0.5 0.09415 = 0.9983 = 0.999152
= 0.9 + 0.99915 = 0.09915 
Problem 9
A 1-year American put option on a stock is modeled with a 2-period binomial tree. You are given:
(i) The initial stock price is 90.
(ii) The strike price is 100.
(iii) The continuous dividend rate is 0.02
(iv) The continuously compounded risk-free interest rate is 5%.
(v) u = 1.2 and d = 0.8.
Estimate annual at the initial node.
Solution.
(S, 0) =

P (Sud) P (S, 0) (S, 0) 0.5(S, 0)2


2h

Here  = Sud S = 3.6, h = 0.5


The stock tree:
u2 S = 129.6
uS = 108
S = 90

udS = 86.4
dS = 72
d2 S = 57.6

The put tree:


Puu = 0
Pu
Pud = 13.6

P
Pd

Pdd = 42.4
Page 7 of 9

Copyright Natalia A. Humphreys, 2014

ACTS 4302. AU 2014. SOLUTION TO HOMEWORK 6.

e(r)h d
e0.030.5 0.8
=
= 0.5378
ud
0.4
1 p = 0.4622
p =

Pu = erh (1 p )Pud = e0.050.5 0.4622 13.6 = 6.1307


Pd = e0.025 (0.5378 13.6 + 0.4622 42.4) = 26.2469
Since 100 72 = 28 > 26.2469, it is optimal to exercise the option. Therefore, we use Pd = 28 in further
calculations.
P = e0.025 (0.5378 6.1307 + 0.4622 28) = 15.8378
Calculating the Greeks, we obtain:


6.1307 28
Pu Pd
eh = e0.020.5
=
= 0.6014
S(u d)
90 0.4


Puu Pud
13.6
= 0.3117
(Su, h) =
eh = e0.01
Suu Sud
43.2


Pud Pdd
28.8
(Sd, h) =
= 0.99
eh = e0.01
Sud Sdd
28.8
0.3117 + 0.99
(Su, h) (Sd, h)
=
= 0.01884
(S, 0) =
S(u d)
36
13.6 15.8378 + 0.6014(3.6) 0.5 0.01884 12.96
=
= 4.5249
1

Problem 10
You are given the following information for two European options on a stock priced using the BlackScholes formula:

50-strike put 55-strike call


Price

0.023

11.327

-0.01

0.85

0.005

0.03

The stock price is 70.


Determine the shares of stock and 55-strike calls one must buy or sell to delta-gamma hedge a sale of
a 50-strike put.
A
B
C
D
E

Buy 0.15 shares of stock and sell 0.17 55-strike calls


Sell 0.17 shares of stock and buy 0.15 55-strike calls
It is impossible to delta-gamma hedge the put because there is not enough information given
Sell 0.15 shares of stock and buy 0.17 55-strike calls
Buy 0.17 shares of stock and sell 0.15 55-strike calls

Key: D
Page 8 of 9

ACTS 4302. AU 2014. SOLUTION TO HOMEWORK 6.

Copyright Natalia A. Humphreys, 2014

Solution. Let x1 be the number of shares of stock purchased or sold and x2 be the number of 55-strike
calls purchased or sold to delta-gamma hedge a sale of a 50-strike put. Then:

x1 + 0.85x2 = 0.01

0.03x2 = 0.005

x2 = 0.005
0.03 = 0.166
x1 = 0.01 0.85 0.166 = 0.15166
Thus, to delta-gamma hedge a sale of a 50-strike put the market-maker will need to sell 0.15 shares of
stock and buy 0.17 55-strike calls for the total investment of 0.15166 70 + 0.166 11.327 = 8.73 .

Page 9 of 9

You might also like